Anda di halaman 1dari 19

C H A P T E R 78 

Pulmonary Embolism and Deep


Vein Thrombosis
Jeffrey A. Kline

PRINCIPLES ultrasound examinations. A two-point venous ultrasound includes


the common femoral and popliteal vein. A three-point ultrasound
This chapter discusses the diagnosis and treatment of venous includes the common femoral, femoral, and popliteal veins. A
thromboembolism (VTE), including deep vein thrombosis (DVT) whole-leg ultrasound includes a three-point ultrasound and the
and pulmonary embolism (PE), from the perspective of the emer- peroneal and tibial calf veins.
gency clinician and provides a resource for the diagnostic consid-
eration and treatment of VTE in the emergency department (ED). Clinical Features

Anatomy and Pathophysiology of Thrombosis Hallmarks of DVT include unilateral limb pain and swelling.
Often, DVT produces initially subtle and nonspecific symptoms,
As illustrated in Fig. 78.1, blood clots form when fibrin produc- such as a mild cramping sensation or sense of fullness in the calf,
tion exceeds its elimination. Factors that enhance fibrinogen without objective swelling on examination. Many patients use the
synthesis and promote its catalysis to fibrin include systemic term Charley horse to describe the sensation of an early DVT.
inflammation, traumatic or immune-related vascular trauma, Because the left iliac vein is vulnerable to compression by the left
inherited thrombophilias and hemoglobinopathies, cancer, preg- iliac artery (May-Thurner syndrome), leg DVT occurs with a
nancy, and sluggish blood flow. The triad of venous injury, slow slightly higher frequency in the left leg compared with the right;
blood flow, and hypercoagulability are the cardinal inciting mech- bilateral leg DVT is found in fewer than 10% of ED patients diag-
anisms for VTE, and most clinical decision rules for VTE incor- nosed with DVT. Similarly, the clinical signs of DVT vary and may
porate these factors. Additionally, each year of life independently include edema, erythema, and warmth of the affected extremity,
increases the likelihood of imbalanced clot formation. Clot for- tenderness to palpation along the distribution of the deep venous
mation can be accelerated by impaired fibrinolysis, as occurs in system, dilation of superficial collateral veins, and a palpable
the metabolic syndrome, and from smoking. venous cord. Fever suggests an alternative diagnosis, such as cel-
DVT represents a disease spectrum ranging from a minimally lulitis. Upper extremity DVT is, by definition, a thrombosis in the
symptomatic isolated calf vein thrombosis to a limb-threatening axillary vein, whereas thrombosis of the brachial vein is a super-
iliofemoral venous obstruction, causing the condition known as ficial thrombosis.
phlegmasia cerulea dolens (Fig. 78.2). In 2011, the Healthcare Usually, upper extremity DVT presents with arm swelling, on
Cost and Utilization Project (HCUP) Nationwide Emergency the same side as an indwelling catheter or recent intravenous infu-
Department Sample (NEDS) had demonstrated that US emer- sion. In the absence of a catheter, the most frequent location of
gency clinicians diagnose lower extremity DVT in approximately arm DVT is on the dominant hand side, and patients may present
170,000 patients, or approximately 1 in every 500 adult ED with a subtle complaint, such as noting that their rings have
patients.1 become tight. Other sites of venous thrombosis occasionally
The venous anatomy of the lower extremity is divided into the encountered in the ED include the jugular, ovarian, mesenteric,
deep and superficial systems (Fig. 78.3). The superficial venous renal, portal, hepatic, cerebral, and retinal veins. These are con-
system consists primarily of the greater and short saphenous veins sidered unusual sites for venous thrombosis.
and perforating veins. The deep venous system includes the ante-
rior tibial, posterior tibial, and peroneal veins, collectively called DIAGNOSIS
the calf veins. The calf veins join together at the knee to form the
popliteal vein, which extends proximally and becomes the femoral Diagnosis of DVT and PE starts with an estimation of the pretest
vein at the adductor canal. The femoral vein was previously probability (PTP). This estimation may be accomplished by the
named the superficial femoral vein but, because this nomencla- clinical gestalt of an experienced practitioner or in conjunction
ture caused dangerous confusion, its use has been abandoned in with a clinical decision tool, such as that derived and validated by
favor of femoral vein. The femoral vein is joined by the deep Wells and colleagues (Table 78.1). PTP for DVT can also be
femoral vein and then the greater saphenous vein to form the assessed by gestalt or an unstructured method with equal accu-
common femoral vein, which subsequently becomes the external racy, although Well’s score may be preferred because it has been
iliac vein at the inguinal ligament. Proximal DVT refers to a clot tested in larger numbers.3 One PTP score has been derived and
in the popliteal vein or higher, whereas distal clot refers to an initially validated for pregnant patients, the LEFt score: 1 point in
isolated calf vein thrombosis. Distal greater saphenous vein clots case of left (L) leg suspicion, 1 point for edema (E), and 1 point
are sometimes denoted as superficial thrombosis, but greater if the suspicion occurred during the first trimester (Ft) of preg-
saphenous clots near its connection with the femoral vein should nancy, with a score of 0 or 1 tantamount to a low PTP.4 The PTP
be referred to and treated as proximal DVT.2 Knowledge of venous dictates the pathway for diagnostic testing (Fig. 78.4). The Wells
anatomy helps practitioners understand the difference in venous and unstructured (gestalt) methods have approximately equal
1051
Descargado para Constanza Vicencio (constanza.vicencio@ua.cl) en Univ Antofagasta de ClinicalKey.es por Elsevier en septiembre 13, 2018.
Para uso personal exclusivamente. No se permiten otros usos sin autorización. Copyright ©2018. Elsevier Inc. Todos los derechos reservados.
1052 PART III  Medicine and Surgery  |  SECTION Four  Vascular System

Older age Exercise


Metabolic syndrome Physical fitness Vena cava interior
Inherited thrombophilia Young age Common iliac vein
Immobility Normative genetics
Cancer No Cancer Internal iliac vein
Non-O blood type O blood type
External iliac vein
Common femoral vein
Coagulation excess and Regulated coagulation Greater saphenous vein
hypofibrinolysis and healthy fibrinolysis
Deep femoral vein
Healthy
blood
flow Femoral vein
Red Thrombus
blood
cells Popliteal vein

Gastrocnemic vein
Fig. 78.1.  Diagram of clotting risk.
Anterior tibial vein
Soleus vein
Peroneal vein
Posterior tibial vein

Fig. 78.3.  Diagram of leg veins. A three-point ultrasound includes the


common femoral, femoral and popliteal veins. A whole-leg ultrasound
adds in the greater saphenous, posterior tibial and peroneal veins and,
in some centers, the gastrocnemius vein.

TABLE 78.1 

Well’s Score for Deep Vein Thrombosis


CLINICAL FEATURE SCORE*
Active cancer (treated within the previous 6 mo or currently 1
receiving palliative treatment)
Fig. 78.2.  Patient with phlegmasia cerulea dolens.
Paralysis, paresis, or recent plaster immobilization of the 1
lower extremities
Recently bedridden for ≥3 days or major surgery within 1
overall diagnostic accuracy.5,6 Either method is acceptable. 12 wk requiring general or regional anesthesia
Although only performed 50% of the time, all patients should
have a PTP assessed and documented prior to additional testing Localized tenderness along the distribution of the deep 1
for DVT or PE.5 For all practical purposes, the diagnosis of DVT venous system
is confirmed by a positive compression ultrasound. Entire leg swollen 1
Calf swelling at least 3 cm larger than on the asymptomatic 1
Differential Diagnosis side (measured 10 cm below the tibial tuberosity)
Venous insufficiency that causes venous hypertension and inflam- Pitting edema confined to the symptomatic leg 1
mation with pain is the most common alternative diagnosis to Collateral superficial veins (nonvaricose) 1
acute DVT, producing many of the same findings (Table 78.2).
Cellulitis is probably the second most common alternative. Previously documented deep vein thrombosis 1
However, in a patient with clinical evidence of cellulitis, the fre- Alternative diagnosis at least as likely as deep vein −2
quency of concurrent DVT is approximately 3%, suggesting that thrombosis
the diagnostic evaluation for DVT in cellulitis patients should *A score <2 indicates that the probability of deep vein thrombosis is low.
be restricted to those with a high PTP.7-9 Other conditions that Adapted from Wells PS, Anderson D, Bormanis J, et al: Value of assessment of
mimic DVT include muscle strain, hematoma, Baker’s cyst, and pretest probability of deep-vein thrombosis in clinical management. Lancet 350:1795–
lymphedema. 1798, 1994.

Descargado para Constanza Vicencio (constanza.vicencio@ua.cl) en Univ Antofagasta de ClinicalKey.es por Elsevier en septiembre 13, 2018.
Para uso personal exclusivamente. No se permiten otros usos sin autorización. Copyright ©2018. Elsevier Inc. Todos los derechos reservados.
C H APTER 78  Pulmonary Embolism and Deep Vein Thrombosis 1053

Assess pretest probability:


Wells score or gestalt assessment

Low Moderate to high


(Wells <2 points (Wells >1 or >15%
or <15%) probability)

Three-point Venous
Quant D-
+ (above calf) ultrasonography
dimer±
ultrasound ± D-dimer

+

Three-point
Whole leg
(above calf)
ultrasound
ultrasound

Three-point ultrasound (–) –


and D-dimer (–)

Repeat venous
ultrasonography – +
In 2-7 days

No DVT DVT (+)

Fig. 78.4.  Diagnostic algorithm to guide the diagnosis and exclusion of acute deep vein thrombosis.
DVT, Deep vein thrombosis; Quant, quantitative; −, test negative; +, test positive.

TABLE 78.2  Diagnostic Testing

Conditions With Symptoms Similar to Deep Laboratory Evaluation


Vein Thrombosis A normal quantitative D-dimer concentration in a patient with a
CONDITION APPROXIMATE FREQUENCY (%) low PTP can be used to exclude proximal DVT (diagnostic sensi-
tivity ≅ 95%; specificity ≅ 50%). The D-dimer results from the
Chronic venous insufficiency* 60 enzymatic breakdown of cross-linked fibrin in any intravascular
Cellulitis 15 thrombus. Many conditions elevate the D-dimer other than DVT
and PE.10 The US Food and Drug Administration (FDA) has
Muscle strain or tear 5
cleared numerous D-dimer assays, and many use varying thresh-
Baker’s cyst 3 old cutoffs for abnormal. Most D-dimer assays that have FDA
Hematoma 2 clearance to aid in the diagnosis and exclusion of VTE have
a cutoff value of 500 ng/mL. Numerous clinical laboratories,
Claudication/ischemia 2 however, use D-dimer assays that do have this specific indication,
Intra-abdominal compression 2 and many of these assays have cutoffs other than 500 ng/mL.
Unrecognized trauma 2
Radiographic Evaluation
*Venous insufficiency causes swelling and venous wall strain, leading to inflammation
and pain. Venous duplex ultrasonography, performed by a certified sonog-
rapher and interpreted by a board-certified radiologist or simi-
larly credentialed expert, has a sensitivity and specificity of
approximately 95%, respectively, for proximal DVT and is the
diagnostic test of choice in most centers. A patient at a low PTP
may have the diagnosis of DVT effectively excluded by a negative

Descargado para Constanza Vicencio (constanza.vicencio@ua.cl) en Univ Antofagasta de ClinicalKey.es por Elsevier en septiembre 13, 2018.
Para uso personal exclusivamente. No se permiten otros usos sin autorización. Copyright ©2018. Elsevier Inc. Todos los derechos reservados.
1054 PART III  Medicine and Surgery  |  SECTION Four  Vascular System

three-point venous duplex ultrasound, which images the common Superficial Leg Thrombophlebitis
femoral, femoral, and popliteal veins (see Fig. 78.2). However, for
patients at higher than low risk, a single negative three-point Based on the results of a large randomized controlled trial, patients
ultrasound is inadequate as a sole method to exclude DVT, with a clot in the greater saphenous vein that extends above the
whereas a single normal whole-leg ultrasound (including normal knee are at risk for progression to DVT via the saphenous-femoral
calf and saphenous veins) is sufficient to exclude DVT with any vein junction and may require an abbreviated course of antico-
PTP.11 A negative three-point ultrasound, together with a negative agulation.17 Published evidence has suggested that distal saphe-
quantitative D-dimer, excludes DVT with any PTP.12 If a patient nous vein thrombophlebitis can adequately be treated with
with a moderate to high PTP and elevated D-dimer level (or not nonsteroidal antiinflammatory drugs, heat, and graded compres-
performed), a negative three-point ultrasound at the index visit sion stockings (fitted to exert 30–40 mm Hg of pressure at the
should be followed by a repeat ultrasound in 2 to 7 days. If nega- ankle), followed by a scheduled repeat ultrasound in 2 to 5 days.
tive, this is sufficient to exclude DVT, and ostensibly, PE. An If a greater saphenous vein clot is proximal, near the connection
expertly performed and interpreted positive ultrasound is suffi- with the femoral vein (see Fig. 78.3), anticoagulation is indicated.
cient to confirm the diagnosis of DVT. Ultrasound cannot be used The precise duration of anticoagulation treatment remains uncer-
to rule out iliac or pelvic vein thrombosis. When duplex ultra- tain, but we recommend full-dose LMW heparin or fondaparinux
sound is not available, patients with a moderate to high PTP for 10 days followed by a repeat ultrasound.18 If the repeat ultra-
should receive empirical low-molecular-weight (LMW) heparin sound shows improvement, anticoagulants can be discontinued.
while awaiting the availability of ultrasound imaging, whereas
patients with a low or moderate to high PTP with a negative Isolated Calf Vein Thrombosis
D-dimer do not need empirical anticoagulation while they wait
for diagnostic imaging.13 Aggregated data have now demonstrated The optimal management strategy for thromboses of the tibial or
that emergency clinician–performed three-point ultrasound for peroneal veins remains controversial, although it is clear that anti-
lower extremity DVT has adequate diagnostic accuracy (96% sen- coagulation lowers the rate of proximal propagation and emboli-
sitivity, 96% specificity) to diagnose and exclude DVT in the zation.19 For tibial or peroneal vein thrombosis in an otherwise
hands of an experienced ultrasonographer.14 healthy ambulatory patient, with no other indications for antico-
Magnetic resonance imaging (MRI) can evaluate the pelvic agulation, the recommendation is short-term anticoagulation,
vasculature and vena cava, which is not possible with ultrasound. most easily accomplished with rivaroxaban (15 mg bid for 14 days
MRI does not produce ionizing radiation. Thus, MRI is a logical then 20 mg QD) or apixaban (10 mg bid for 7 days, then 5 mg
option to evaluate the pelvic veins of patients at high risk for bid for 7 days), or antiplatelet therapy with aspirin (325 mg/day
pelvic vein thrombosis (eg, those with gynecologic malignancy) of enteric-coated acetylsalicylic acid) and close follow-up with
and for pregnant patients. Its use is limited by cost, availability, repeat duplex ultrasound scan at 2 to 5 days to evaluate for clot
patient size, and tolerance to close quarters. MRI is not the propagation.
primary diagnostic test for patients with suspected DVT.
Phlegmasia Cerulea Dolens (Painful Blue Leg)
Management
Massive iliofemoral vein occlusion results in swelling of the entire
For patients with high PTP after hours, and for patients with a leg, with extensive vascular congestion and associated venous
positive ultrasound, anticoagulation should be initiated emer- ischemia, producing a painful cyanotic extremity. There may be
gently, unless contraindicated, as outlined in Table 78.3. Most an associated arterial spasm resulting in phlegmasia alba dolens
patients with DVT can be treated at home, assuming that the (painful white leg or so-called milk leg), which may mimic an
patient can effectively adhere to the chosen anticoagulation strat- acute arterial occlusion. Prompt consultation with a vascular
egy. The antiquated concept that patients with DVT should be at surgeon should be obtained because patients with phlegmasia
bed rest is categorically incorrect, and patients should be encour- cerulea dolens may require emergent thrombectomy. If timely
aged to ambulate after anticoagulation for DVT to reduce the consultation is not possible, early thrombolytic therapy may be a
incidence of postthrombotic syndrome. Note that the presence of limb-salvaging procedure in the absence of contraindications.
a so-called free-floating DVT does not increase risk of emboliza- One strategy is to infuse alteplase via an infusion catheter placed
tion.15 Compression stockings can no longer be advocated rou- into the thrombus. This procedure requires interventional radiol-
tinely for DVT, although patients with persistent swelling or ogy capabilities, and therefore emergency clinicians caring for
superficial thrombosis may benefit.16 patients with evidence of phlegmasia cerulean dolens in hospitals

TABLE 78.3 

Emergency Department Anticoagulation for Deep Vein Thrombosis or Pulmonary Embolism


ANTICOAGULANT INITIAL DOSE RESTRICTION TIME TO PEAK (H)
Unfractionated heparin 70–80 U/kg, then 17–18 U/kg/h, IV Heparin-induced thrombocytopenia 1
Enoxaparin 1 mg/kg subcutaneously* Creatinine clearance < 30 mL/min 3
Dalteparin 200 U/kg subcutaneously* Creatinine clearance < 30 mL/min 4
Fondaparinux 5−10 mg subcutaneously* Creatinine clearance < 30 ml/min 3
Rivaroxaban 15 mg orally with food Creatinine clearance < 30 mL/min 2−4
Apixaban 10 mg orally, with or without food Creatinine clearance < 30 mL/min 3−4
*Although low-molecular-weight heparin compounds are usually injected subcutaneously, no trials have been conducted to justify this route over intravenous injection. Intra-
venous injection achieves more rapid anticoagulation and does not produce more bleeding.

Descargado para Constanza Vicencio (constanza.vicencio@ua.cl) en Univ Antofagasta de ClinicalKey.es por Elsevier en septiembre 13, 2018.
Para uso personal exclusivamente. No se permiten otros usos sin autorización. Copyright ©2018. Elsevier Inc. Todos los derechos reservados.
C H APTER 78  Pulmonary Embolism and Deep Vein Thrombosis 1055

without the resources immediately available should not delay on the job, impairing his ability to work. Compression stockings
transfer to an interventional radiology–capable center. reduced the swelling and provided some improvement.

Upper Extremity Venous Thromboses Disposition


DVTs of the upper extremity have become more common in Assuming that systemic anticoagulation can be reliably estab-
association with the increased use of indwelling venous catheters lished, most patients with acute DVT can be discharged from the
and wires for electronic cardiac devices. Upper extremity DVT can ED. Protocols that use monotherapy such as apixaban or rivaroxa-
cause PE, and all patients with DVT above the elbow require ban can facilitate this process.26 I recommend selecting patients
definitive treatment.: About half of all upper extremity DVTs are for home therapy using the modified Hestia criteria (Box 78.1).
associated with an indwelling catheter, and peripherally inserted
central catheters (PICCs) carry the highest risk.20,21 Aggregated PULMONARY EMBOLISM
data have indicated that only venous ultrasound has been ade-
quately validated as a method to diagnose and exclude upper Principles
extremity DVT, and D-dimer has only been examined in one
study.22 In the absence of pain or infection, catheter-associated A PE results when a clot that formed hours, days, or sometimes
DVT does not automatically warrant catheter removal if the cath- weeks earlier in the deep veins dislodges, travels through the
eter serves a current and vital purpose. However, these patients venous system, and traverses the right ventricle into the pulmo-
should receive anticoagulation absent contraindications. The nary vasculature.
duration of recommended anticoagulation following catheter
removal for DVT remains variable, but most published guidelines Pathophysiology of Pulmonary Vascular Occlusion
recommend at least 3 months.23 Acute PE from an axillary vein
occurs in about 9% of patients with arm DVT, although the PE The right ventricle normally pumps through a pulmonary vascu-
tends to be less severe from upper extremity DVT.24 Isolated upper lar tree with a low resistance to fluid flow, and young persons
extremity DVT, especially axillary-subclavian vein thrombosis, without cardiopulmonary disease (eg, congestive heart failure,
also can be seen in relatively young, active, otherwise healthy chronic obstructive lung disease, advanced sarcoidosis, pulmo-
patients after considerable exertion of the dominant arm, known nary fibrosis, scleroderma, primary pulmonary hypertension) can
as effort DVT, or Paget-Schroetter syndrome. Optimal treatment tolerate at least 30% obstruction from a clot, with minimal symp-
of isolated brachial vein thrombosis, often the result of a recent toms or signs. Pulmonary infarction, in contrast, can produce
intravenous infusion (so-called infusion phlebitis) also remains severe pleuritic pain. Although a segmental pulmonary artery
uncertain, and no study has demonstrated clear benefit for sys- constitutes only about 116 of the entire pulmonary vascular circuit,
temic anticoagulation. I recommend the same management plan a clot lodged deeply in a segmental artery can obstruct blood flow
as described for superficial thrombophlebitis of the leg. to a sufficient degree to cause tissue necrosis.
Table 78.4 presents a listing of factors that significantly increase
Complications the probability of PE in the ED population.27 Not all variables that
increase the probability of PE in epidemiologic studies also
Although the most feared complication of DVT is fatal PE, DVT increase the probability of a PE diagnosis in individual ED patients
damages venous valves, causing venous insufficiency. Venous with signs and symptoms suggesting PE. From an epidemiologic
insufficiency, in turn, manifests as a spectrum ranging from pain- standpoint, people who smoke have a significantly higher risk for
less varicosities to severe postthrombotic syndrome, which can venous clots than people who do not smoke. However, in the ED,
cause unremitting pain and swelling, varicose veins, skin changes, smoking does not seem to increase that person’s risk for PE over
and nonhealing ulcers in 5% to 10% of patients.25 Fig. 78.5 shows that of a nonsmoker with an otherwise identical clinical presenta-
the leg of a construction worker with a femoral DVT that pro- tion. It is possible that smokers are simply more likely to have
duced moderate postthrombotic syndrome, resulting in swelling other lung problems that manifest a clinical presentation similar
to that of PE. As many as 50% of patients diagnosed with PE have
no apparent clinical risk factors for VTE, but testing for genetic

BOX 78.1 

Modified Hestia Criteria to Select Patients With


Deep Vein Thrombosis and/or Pulmonary
Embolism for Outpatient Treatment
Identifies low-risk PE if:
• Systolic blood pressure > 100 mm Hg
• No thrombolysis needed
• No active bleeding
• Oxygen required to maintain oxygen saturation > 94%
• Not already anticoagulated
• Absence of severe pain requiring > two doses of intravenous
narcotics
• Other medical or social reasons to admit
• Creatinine clearance > 30mL/min
Fig. 78.5.  Patient with moderate postthrombophlebitic syndrome in the • Not pregnant, severe liver disease, or heparin-induced
left leg several months after diagnosis with a common femoral DVT. thrombocytopenia
Observe the swollen appearance and slight color change in the foot.

Descargado para Constanza Vicencio (constanza.vicencio@ua.cl) en Univ Antofagasta de ClinicalKey.es por Elsevier en septiembre 13, 2018.
Para uso personal exclusivamente. No se permiten otros usos sin autorización. Copyright ©2018. Elsevier Inc. Todos los derechos reservados.
1056 PART III  Medicine and Surgery  |  SECTION Four  Vascular System

TABLE 78.4 

Evaluation of Classic Risk Factors and Physiologic Findings for Pulmonary Embolism in the Emergency
Department (ED) Setting
STRENGTH OF ASSOCIATION WITH
FACTOR MECHANISMS PE DIAGNOSIS IN ED POPULATIONS
Inherited thrombophilia Hypercoagulability ++
Connective tissue disease Inflammation Unknown
Acquired thrombophilia Hypercoagulability Unknown
Active cancer (under treatment) Hypercoagulability ++
Inactive cancer (considered in remission) Presumed hypercoagulability Not significant
Limb or generalized immobility Stasis ++
Recent travel Stasis Minimal
Prior PE or DVT Multiple +
Trauma within past 4 wk requiring hospitalization Inflammation, venous injury and stasis +++
Surgery within past 4 wk requiring general anesthesia Inflammation, venous injury and stasis ++++
Smoking Inflammation and hypofibrinolysis Not significant
Estrogen Hypercoagulability ++
Pregnancy, postpartum Hypercoagulability Minimal
Family history of VTE Inherited condition Not significant
SYMPTOMS
Pleuritic chest pain Lung ischemia, muscle strain +
Substernal chest pain Presumed cardiac ischemia Not significant
Dyspnea   mismatch
V/Q +
Sudden onset of symptoms Vascular obstruction Not significant
Hemoptysis Infarction +++
Syncope Vascular obstruction Minimal
SIGNS
Pulse rate > 100 beats/min Cardiac stress, baroreceptors ++
Pulse oximetry reading < 95% (sea level)   mismatch
V/Q +++
Unilateral leg or arm swelling Venous obstruction ++++
Normalization of vital signs Presumptive from treatment or Hawthorne effect Not significant
  , ventilation−perfusion ratio.
DVT, Deep vein thrombosis; PE, pulmonary embolism; V/Q

thrombophilia has no value in the ED setting, or any other Pulmonary embolism can produce hypoxemia (pulse oximetry
setting.28 reading <95% at sea level or <92% in Denver or Salt Lake City),
but the degree of hypoxemia is unpredictable. Approximately half
Clinical Features of all patients with PE have no evidence of hypoxemia. A swine
model mimicking massive pulmonary vascular occlusion (increase
Symptoms vary widely during this process, ranging from no in systolic pulmonary arterial pressure to ≈65 mm Hg) did not
symptom to cardiovascular collapse. The patient can feel focal, show any decrease in pulse oximetry reading (from 98% preem-
sharp, pleuritic pain and exhibit a splinting response to breathing. bolization to 98% postembolization).30 Despite its shortcomings
Over several days, the infarcted segment becomes consolidated on as a single diagnostic step, the presence of hypoxemia (pulse oxim-
chest radiography and exudes a pleural effusion, manifesting an etry <95%, breathing room air) that cannot be explained by a
intense underlying inflammatory process. Chest pain from non- known disease process increases the probability of PE. Conversely,
infarcting PE can be highly variable and vague, with as many as a normal oxygen saturation, although reassuring, cannot rule out
30% of patients with definite PE having no perception of chest PE. When PE is diagnosed, the severity of hypoxemia represents
pain.29 a significant independent predictor of patient outcome.
In contrast, if asked in a detailed and structured way, approxi- PE also causes highly variable effects on other vital signs. In
mately 80% of patients with PE admit to having some sensation the ED, about half of all patients with PE have a heart rate greater
of dyspnea.29 The dyspnea may be constant and oppressive or may than 100 beats/min.27 Tachycardia from PE probably results from
be intermittent and perceived only with exertion, possibly due to impaired left ventricular filling, leading to a pathophysiologic
an exercise-induced increase in pulmonary vascular resistance. process that parallels that of hemorrhagic shock. Only about half

Descargado para Constanza Vicencio (constanza.vicencio@ua.cl) en Univ Antofagasta de ClinicalKey.es por Elsevier en septiembre 13, 2018.
Para uso personal exclusivamente. No se permiten otros usos sin autorización. Copyright ©2018. Elsevier Inc. Todos los derechos reservados.
C H APTER 78  Pulmonary Embolism and Deep Vein Thrombosis 1057

BOX 78.2 

Pulmonary Embolism Rule-Out Criteria


(PERC Rule)
Low pretest probability for PE by the treating emergency clinician’s
unstructured estimate, plus all the following must be true:
• Age < 50 yr
• Pulse rate < 100 beats/min
• Oxygen saturation > 94%
• No hemoptysis
• No unilateral leg swelling
• No recent major surgery or trauma
• No prior pulmonary embolism or deep venous thrombosis
• No hormone use
Fig. 78.6.  Massive pulmonary embolism on autopsy. This man died with PE, Pulmonary embolism.
a large clot burden that plugged the distal lobar arterial branches, eventu-
ally producing nearly complete obstruction to blood flow and subsequent
cardiac arrest. He had vague respiratory symptoms for 2 weeks, causing
him to see a physician, who diagnosed bronchitis. pneumonia, including focal chest pain, fever, and unilateral rales
on auscultation. However, a temperature greater than 101.5°F
(38.6°C) suggests infection rather than infarction. On occasion,
pulmonary infarction may present with an onset of pain and
of patients have an elevated respiratory rate (>20 breaths/min). hemoptysis simultaneously. In contrast, lobar pneumonia, which
The probability of PE was not reduced in patients who normalized usually presents with productive cough for a few days before rust-
any vital sign while in the ED.31 About half of patients with PE tinged sputum appears. Isolated substernal chest pain is a rare
have a dilated RV on an echocardiogram obtained in the ED.32 presentation for PE and, in general, suggests a cardiac or other
Arterial hypotension (systolic blood pressure < 90 mm Hg) rep- origin.
resents an ominous hemodynamic consequence of PE; it occurs Most patients with PE have no obvious abnormality on physi-
in only about 10% of patients, but signifies a fourfold increase cal examination, other than an affect that gives the appearance of
in risk of death compared with normotensive patients.33 In its distress or anxiousness, with respiratory distress.35 The only posi-
most extreme form, PE can obstruct the right ventricular outflow tive finding from the itemized physical examination that reliably
entirely by casting the entire pulmonary vascular tree (Fig. 78.6) increases the probability of PE is evidence of a DVT—unilateral
or acutely occluding the main pulmonary artery. Pulseless electri- leg asymmetry, unilateral edema, tenderness along a deep vein. On
cal activity (PEA) is the most common electrocardiographic result the other hand, wheezing, or a prolonged expiratory phase on
from obstructive PE. The survival rate from cardiac arrest from lung auscultation, suggests the alternative diagnosis of broncho-
PE is about 20%, even if the arrest is witnessed, and treatment spasm, which reduces the probability of PE. Bilateral rales suggest
with bolus fibrinolysis is initiated.34 the diagnosis of left ventricular failure, although localized rales
PE can present as cardiac arrest. Most patients with incipiently often are heard over infarcted lung tissue.
fatal PE have overt respiratory distress, syncope or seizure-like When retrospectively comparing patients in whom the diag-
activity, or high heart rate relative to the systolic blood pressure nosis of PE was delayed, patients who were admitted to the hos-
before arrest. First responders who observe a patient dying from pital tended to have a higher frequency of altered mental status,
PE usually observe PEA as the initial cardiac arrest rhythm (>20 new or at baseline dementia, and more comorbid conditions.36,37
depolarizations/min, without palpable pulses). The mechanism Only one recent study has evaluated patients discharged and sub-
for PEA manifests from right ventricular outflow obstruction and sequently diagnosed with PE. Those patients tended to lack
impaired right ventricular contractility. Ultrasound performed dyspnea and had pleuritic chest pain and hemoptysis, together
during PEA arrest from PE usually shows weak cardiac contrac- with a pulmonary infiltrate on imaging, lower D-dimer concen-
tions, with a swollen right ventricle and small left ventricle. Some tration, and small distal clot seen on pulmonary vascular imaging.
patients manifest slow agonal rhythms with fatal or near-fatal PE, Thus, patients who were discharged with PE seemed to have iso-
possibly due to septal wall tension leading to ischemia or an lated pulmonary infarction that was often misidentified as pneu-
ischemic-equivalent effect on the atrioventricular node and monia. Coincidentally, in a secondary analysis of a large database
infranodal conducting pathways. of PE-positive patients, to which I applied the PERC (pulmonary
Virtually any ED visit related to weakness, shortness of breath, embolism rule-out criteria) rule (Box 78.2) to understand the
dizziness or syncope, pain, extremity discomfort, or nonspecific profile of the PE-positive but PERC-negative patients better, the
malaise or functional deterioration could represent a potential PE. presence of pleuritic chest pain emerged as a common feature.38
However, this does not mean that every patient with these symp- It appears as though emergency clinicians may be prone to miss
toms should be evaluated for PE, and these symptoms need to be small, distal lung clots that produce a clinical picture of pneumo-
rationally considered in the context of the entire clinical picture. nia. More evidence is needed to determine if these patients, in the
A patient with PE typically presents with 2 to 3 days of constant absence of DVT, benefit from systemic anticoagulation.
or worsening shortness of breath. For many patients, the dyspnea
is only present with exertion, and patients often need to be Differential Diagnosis
prompted to endorse this symptom. Patients usually describe
chest pain with PE in vague terms, unless they have pulmonary Pneumonia is the most common alternative diagnosis found in
infarction. About 20% of ED patients with PE have focal pleuritic ED patients, diagnosed in 5% to 10% of scans, and in many
chest pain, but many say nonspecifically that their chest hurts with studies was a more common finding than PE on computed
breathing, usually on the lateral aspects. Those with lung infarc- tomography pulmonary angiography (CTPA).39 Other similar
tion can present with a clinical picture similar to that of lobar findings include exacerbations of chronic obstructive pulmonary

Descargado para Constanza Vicencio (constanza.vicencio@ua.cl) en Univ Antofagasta de ClinicalKey.es por Elsevier en septiembre 13, 2018.
Para uso personal exclusivamente. No se permiten otros usos sin autorización. Copyright ©2018. Elsevier Inc. Todos los derechos reservados.
1058 PART III  Medicine and Surgery  |  SECTION Four  Vascular System

disease, asthma, pulmonary vascular disease, including all causes the so-called Hampton’s hump finding. Unilateral lung oligemia
of pulmonary hypertension, pericarditis, pleurisy, costochon- (Westermark’s sign) is a rare radiographic manifestation of a
dritis, spontaneous pneumothorax, acute coronary syndrome large PE.
(ACS), and chest wall trauma. Most alternative diagnoses can be Likewise, a 12-lead ECG provides more information about the
ruled out with a thorough history, physical, chest x-ray, electro- presence of alternative diagnoses (eg, pericarditis, cardiac isch-
cardiogram (ECG), cardiac enzyme testing, and echocardiogra- emia) than the presence of PE. When PE causes electrocardio-
phy.40,41 When the diagnosis is unclear, consider observation or graphic changes, this is usually a result of acute or subacute
admission. pulmonary hypertension. The most common effects of pulmo-
nary hypertension on the ECG are rapid heart rate, symmetric
Diagnostic Testing T-wave inversion in the anterior leads (V1–V4), the McGinn-
White S1Q3T3 pattern, and incomplete or complete right bundle
Fig. 78.7 illustrates an algorithmic approach to PE exclusion and branch block (Fig. 78.8). Any one of these findings approximately
diagnosis in nonpregnant patients. Chest radiography seldom doubles the probability of PE in a symptomatic patient.
provides specific information, but is useful to suggest alternative In the ED, inability to identify a cause of chest symptoms or
diagnoses, such as pneumonia, congestive heart failure, or pneu- specific signs may be an important cue to evaluate the patient for
mothorax. If symptoms have been present for 3 days or more, a PE. Because as many as 50% of patients diagnosed with PE have
pulmonary infarction may be visible on chest x-ray as an apex- no identifiable classic risk factors for thrombosis, the decision to
central, pleural-based, wedge-shaped area of infiltrate, producing pursue the diagnosis of PE is based on that particular patient’s

Assess pretest probability:


Gestalt or
Wells score or
revised Geneva score (RGS)

High (>40%)
Low Moderate (15-40%) or
(<15%) or Wells >4
Wells ≤4 or
or sRGS >4
sRGS ≤4
PERC rule
Consider LMWH if no
contraindication

– +
Order imaging

Quant D-
No PE
dimer*
Serum Cr→ Serum Cr→
GFR <60 GFR >59
mL/min mL/min
<500 ≥500
ng/mL ng/mL
Or < age*10 Or ≥ age*10
V/Q CTPA
ng/mL ng/mL

NI High Nondx
+ –

No PE PE (+)

Fig. 78.7.  Suggested algorithm to evaluate for pulmonary embolism (PE) in the emergency department
(ED). This algorithm include the use pretest probability (PTP), enzyme-linked immunosorbent assay (ELISA)
or immunoturbidometric quantitative D-dimer assay, and pulmonary vascular imaging (CT pulmonary
angiography [CTPA]). The algorithm recommends V/Q scanning for patients with compromised renal func-
tion, defined by an estimated glomerular filtration rate (GFR) <60 mL/min. *Assumes a standard cutoff
for normal of 500 ng/mL for D-dimer. Patients with a high pretest probability and negative CTPA or non-
diagnostic V/Q scanning may require additional testing—lower extremity venous ultrasound and, if initial
venous ultrasound is negative, repeat the lower extremity venous ultrasound in 1 week. CXR, Chest x-ray;
Cr, creatinine; GFR, glomerular filtration rate; High, high probability scan findings; LMWH, low-molecular-
weight heparin; Nl, normal; Nondx, nondiagnostic (any reading other than normal or high probability);
PERC, pulmonary embolism rule-out criteria; quant, quantitative, sRGS, simplified revised Geneva score;
+, positive for PE; −, negative for PE.

Descargado para Constanza Vicencio (constanza.vicencio@ua.cl) en Univ Antofagasta de ClinicalKey.es por Elsevier en septiembre 13, 2018.
Para uso personal exclusivamente. No se permiten otros usos sin autorización. Copyright ©2018. Elsevier Inc. Todos los derechos reservados.
C H APTER 78  Pulmonary Embolism and Deep Vein Thrombosis 1059

I aVR V1 V4

II aVL V2 V5

III aVF V3 V6

VI

II

V5

Fig. 78.8.  Initial electrocardiogram tracing from an 18-year-old woman on oral contraceptives who
presented to the ED with syncope. Several findings consistent with pulmonary embolism are shown,
including tachycardia, early S1Q3T3 pattern, and incomplete right bundle branch block. CT angiography
revealed extensive bilateral pulmonary emboli, and echocardiography showed severe right heart
dysfunction.

presentation and should not rely on the presence or absence of Evaluation for PE begins with an assessment of PTP and
epidemiologic risk factors. patients without symptoms or signs of PE (eg, no chest pain, no
In some cases, PE can be excluded with reasonable certainty shortness of breath, no dyspnea on exertion, normal vital signs,
based on data available at the bedside, gathered only by the and no recent syncope) should not be tested, even in the presence
medical history and physical examination. Multicenter studies of of risk factors. Moreover, many patients with a risk factor and a
urban academic EDs have suggested that emergency clinicians symptom or sign of PE can still have PE safely excluded without
currently evaluate approximately 2% of all patients for PE with diagnostic testing. Because the evaluation for PE relies heavily on
CTPA.42,43 Each year, more than 16 million patients, or 12% of all the PTP, an important question to answer is how to quantify the
patients who present to the ED, have chest pain or dyspnea, and PTP accurately. Several clinical decision rules have been derived
not all require an evaluation for PE. Although numerous cases of and validated for the risk stratification of patients with possible
PE are likely still missed, overtesting for PE can also be harmful. PE; however, difficulty with spontaneous recall and a preference
Specific risks include exposure to the ionizing radiation and IV for gestalt reasoning by clinicians may limit their use in clinical
contrast necessary for CTPA and the risk of a false-positive inter- practice. Fortunately, gestalt reasoning appears to be comparable
pretation, which may occur in as many as 10% of scans read as to other validated decision rules.45,47
positive for PE.44 The appropriate use of D-dimer testing decreases Although gestalt reasoning and clinical decision rules may
the need for imaging in all patients with non–high PTP. provide adequate stratification to guide the evaluation (ie,
A rational strategy to evaluate a patient for PE should begin D-dimer vs. pulmonary vascular imaging), these methods alone
with estimation of the PTP for PE. Methods for estimating PTP do not reproducibly identify the very low-risk population whose
can be implicit, the clinician’s gestalt best guess, or explicit—use PTP lies below the 2% test threshold. To identify the very low-risk
of a scoring system, which is synonymous with a clinical decision group in whom PE could be safely excluded at the bedside,
rule, or clinical prediction rule to categorize the probability (eg, with no diagnostic testing, the PE rule-out criteria (or PERC
Well’s score, Geneva criteria, Charlotte rule).45 rule; see Box 78.2) can be used.48 When the physician’s unstruc-
One approach to the evaluation for PE is to compare the PTP tured clinical suspicion for PE is low, and each of the eight ele-
with the test threshold for PE. The test threshold represents the ments of the rule is satisfied, the PERC rule identifies a very
point above which some type of evaluation should be initiated low-risk population among whom no patient has a PTP for PE
and below which the clinician can justify not starting the evalua- greater than 2% and obviates further testing in about 20% of ED
tion. For PE, the test threshold is from 1% to 5%.46 I recommend patients.48
that patients with a a PTP less than approximately 2% are more For a patient with a high PTP (by any method), emergency
likely to be harmed than benefited by an evaluation and vice versa clinicians should order pulmonary vascular imaging and consider
for patients with a PTP greater than 2%. Thus some patients with initiating anticoagulation in the absence of contraindications.49
symptoms and signs of PE can have PE excluded at the bedside Patients with a non–high PTP (simplified revised Geneva score <
using the combination of PTP and additional explicit criteria. 5, Wells score < 5, or gestalt PTP < 40%) can have PE excluded
Other patients require additional objective diagnostic testing. with a normal D-dimer concentration, using the cutoff for

Descargado para Constanza Vicencio (constanza.vicencio@ua.cl) en Univ Antofagasta de ClinicalKey.es por Elsevier en septiembre 13, 2018.
Para uso personal exclusivamente. No se permiten otros usos sin autorización. Copyright ©2018. Elsevier Inc. Todos los derechos reservados.
1060 PART III  Medicine and Surgery  |  SECTION Four  Vascular System

abnormal established by the local laboratorian. Additionally,


assuming the D-dimer of use has a cutoff for abnormal of 500 ng/
mL, PE can be excluded with a D-dimer concentration elevated
on the basis of age using the following:

Age ×10 ng/mL

Thus, an 80-year-old patient with a PE unlikely or non–high PTP


can have PE excluded with a D-dimer concentration less than
800 ng/mL.50,51 This strategy maintains a diagnostic sensitivity
near 95% but increases the percentage of patients who can have
PE excluded without pulmonary vascular imaging. The safety of
this strategy has not been tested with D-dimer assays with abnor-
mal thresholds different than 500 ng/mL.
The most common causes of a false-negative D-dimer are very
small isolated subsegmental PE and chronic PE.39 Because the
half-life of circulating D-dimer is less than 8 hours, the sensitivity
of the D-dimer may decrease if the patient’s symptoms have been
present for longer than 3 days. False-negative D-dimer measure- Fig. 78.9.  Scintillation (99Tc) ventilation–perfusion lung scan images
ments may also be seen with severe lipemia and ongoing warfarin showing high probability of acute pulmonary embolism using criteria
therapy. defined in the prospective investigation of pulmonary embolism diagnosis
When the PTP is high, or the screening D-dimer is positive, (PIOPED). The first and third rows project the perfusion phases of the
pulmonary vascular imaging by CTPA or V/Q   scanning is advised. examination, and the second and fourth rows show the ventilation
Although CTPA is not perfect, it has multiple advantages over phases. The black arrowheads point to wedge-shaped defects in the
  scanning and usually can confirm or exclude the presence of perfusion images. Comparison with the corresponding ventilation view
V/Q immediately below shows relatively homogeneous scintillation activity in
PE. the anatomic segments that lack perfusion. These defects are consistent
Most academic centers now use CTPA as the primary method with the effect of acute pulmonary embolism.
of evaluating for PE. The diagnostic sensitivity and specificity of
a technically adequate CTPA scan, performed on a multidetector
row scanner, are both about 90%.52 A good-quality CTPA scan Pregnant Women
offers the highest level of diagnostic and exclusionary certainty
for acute PE. Technical adequacy requires more than 200 HU of Fig. 78.10 presents an algorithm to diagnose PE in pregnant
contrast opacification in the main pulmonary artery and absence patients. It is prudent to explain the diagnostic options to the
of motion artifact. Emergency clinicians should consult with the patient, including the risks and benefits of the various tests, obtain
radiologist interpreting the CTPA scan to ensure good-quality her preferences, and document these stated preferences. The algo-
images for patients with a negative result but a high PTP. If the rithm starts with bilateral lower extremity venous ultrasound. If
radiologist indicates that the scan was significantly compromised, the bilateral ultrasound is positive, treatment can be started. Oth-
the suggested next step is to perform bilateral lower leg ultraso- erwise, the next step is determined by PTP assessment. To my
nography and, if negative, repeat bilateral leg ultrasonography 2 knowledge, no PTP rules have been validated in pregnant patients.
to 7 days later. It is clear that over half of all VTE cases diagnosed in pregnancy
Emergency clinicians have observed an increased detection of occur in the third trimester.55 Based on available patient level data
isolated subsegmental filling defects on chest CT, revealed with from pregnant ED patients, high Wells and Geneva scores, the
more thinly collimated images acquired with multidetector row third trimester, or unexplained hypoxemia (Sao2 < 95% breathing
scanner technology.43 When two radiologists independently eval- room air at sea level) predict a relatively higher PTP for PE.56 Most
uate CTPA, their agreement on the presence of isolated subseg- patients with pregnancy selected by emergency clinicians for PE
mental filling defects is poor.53 evaluation have a low clinical probability.56 Specialty groups have
CTPA can provide additional information to enhance its use- acknowledged the lack of evidence regarding a recommended
fulness in the ED. Although the scan can be extended to include approach to the evaluation of pregnant patients with suspected
the leg veins (CT venography), the technical reliability of this PE, but all agree that effort should be made to avoid fetal exposure
technique has been questioned, causing it to be abandoned in to radiation and iodinated contrast.57,58 To minimize radiation
most centers.54 CTPA often provides information about alterna- exposure, I propose a combined approach, in which negative bilat-
tive processes that might explain the patient’s symptoms. eral lower extremity venous ultrasonography is supported by a
  scintillation scan, introduced in the early 1960s,
The V/Q negative PERC rule and a threshold-adjusted D-dimer assay. The
remains a viable diagnostic option for patients with contraindica- D-dimer threshold can be adjusted according to the trimester of
tions to iodinated intravenous contrast and vulnerable kidney pregnancy, as follows: first trimester, 750 ng/mL; second trimester,
function. The accuracy and precision of the V/Q  scan were shown 1000 ng/mL; third trimester, 1250 ng/mL.59-61 If the patient is has
in the Prospective Investigation of Pulmonary Embolism Diagno- a non–high PTP, has no high-risk factors, is PERC-negative, the
sis (PIOPED) study, which compared the results of V/Q   scanning bilateral ultrasound is negative, and the D-dimer is below the
with the most accurate criterion standard test available at the time, trimester-adjusted values, PE can be excluded. Note that this rec-
formal pulmonary angiography. Fig. 78.9 shows the results of a ommendation does not state that the PERC criteria can be used
high- probability V/Q scan. A high-probability V/Q  scan can con- alone in pregnancy.
 
firms PE, and a normal V/Q scan (ie, no perfusion defect) excludes If the D-dimer is abnormal or the patient fails the PERC cri-
PE. A moderate-probability or indeterminate scan is essentially teria, a pulmonary vascular imaging study is warranted. The best
nondiagnostic and requires additional formal pulmonary angiog- choice of pulmonary vascular imaging is controversial and uncer-
raphy or CTPA. Patients with a low PTP and a low-probability tain.57 Current data indicate that CTPA or V/Q   scanning will
  scan require additional testing, CTPA or bilateral venous
V/Q produce adequate images to exclude and diagnose PE in a preg-
duplex ultrasonography of the legs. nant patient.58,62,63 The data used to estimate the risk of fetal

Descargado para Constanza Vicencio (constanza.vicencio@ua.cl) en Univ Antofagasta de ClinicalKey.es por Elsevier en septiembre 13, 2018.
Para uso personal exclusivamente. No se permiten otros usos sin autorización. Copyright ©2018. Elsevier Inc. Todos los derechos reservados.
C H APTER 78  Pulmonary Embolism and Deep Vein Thrombosis 1061

Pregnant with
suspected PE

Bilateral lower
extremity venous
ultrasound (full leg
preferable)

– +

Non-high PTP, PTP high (>40%), Wells > 4,


first or second sRGS > 4, or presence of other high
trimester risk factors (eg, unexplained
hypoxemia, third trimester)

PERC rule +

Option 1 Option 2

Shared decision- CXR Abnormal


making: Quant D-
dimer*
Q or VQ +
Normal CTPA
scan

1st 2nd 3rd


<750 <1000 <1250 –
NI High Nondx

Yes No Begin
LMWH

Option 1 or 2

No PE

Fig. 78.10.  Algorithm for exclusion and diagnosis of PE in pregnant patients. Shared decision making
refers to discussion of the diagnostic options with the patient, including risks of potentially fatal PE if not
diagnosed, and the potential risks of CTPA or V/Q scanning to the fetus. Non–high PTP refers to the
absence of high PTP by gestalt, Wells, or sRGS. See text for references. *D-dimer concentrations per tri-
mester given in ng/mL assuming a standard D-dimer threshold for abnormal of 500 ng/mL. CXR, Chest
x-ray; Cr, creatinine; CTPA, CT pulmonary angiography; High, high probability scan findings; LMWH, low-
molecular-weight heparin; Nl, normal; Nondx, nondiagnostic (any reading other than normal or high
probability); PERC, pulmonary embolism rule-out criteria; Q, perfusion lung scan; quant, quantitative, sRGS,
simplified revised Geneva score; +, positive for PE; −, negative for PE.

exposure to radiation for CT scanning versus V/Q   scanning are perfusion lung scan is not normal, and CT scanning is ultimately
highly speculative. Shielding the abdomen with a lead or bismuth- required, the mother and fetus will be exposed to more radiation
antimony apron during CT scanning may reduce radiation based than if CTPA had been performed first.
on phantom modeling.64 When available, tube voltage modulating
technology may also serve to lower fetal radiation exposure more Management
than shielding.64 However, if both tests are equally available, I
prefer to consult with the radiologist on duty to coordinate a Fig. 78.11 presents a comprehensive management plan for diag-
stepwise evaluation of the chest radiograph first and, if normal, nosed PE relevant to the context of a large, full- service (typically
to proceed to perfusion-only nuclear lung scanning with a half- known as a tertiary care) hospital. Pathways similar to this have
dose 99Tc-macroaggregate. Because 99Tc is excreted in the urine, been adopted by multidisciplinary PE response teams.65 At the
prehydration with 1 L of intravenous saline and insertion of a left-most side of the algorithm, patients can be discharged to
Foley catheter appears to be a logical but unproven step to reduce home from the ED. At the right side, patients with a massive PE
fetal exposure to radiation. The risk of this approach is that if the and no contraindications receive bolus thrombolytic therapy.

Descargado para Constanza Vicencio (constanza.vicencio@ua.cl) en Univ Antofagasta de ClinicalKey.es por Elsevier en septiembre 13, 2018.
Para uso personal exclusivamente. No se permiten otros usos sin autorización. Copyright ©2018. Elsevier Inc. Todos los derechos reservados.
1062 PART III  Medicine and Surgery  |  SECTION Four  Vascular System

Postive CT pulmonary
angiography or high-
probability V/Q scan

Risk-stratify:
Vital signs, CTPA findings,
BNP, troponin, transthoracic
echocardiography1

Submassive with RV strain: Massive PE: Proximal clot,


Least severe PE:
Normotensive, proximal clot and: SBP < 90 mm Hg for >15 min
Normotensive
RV>LV on CTPA, abnormal echo or 40 mm Hg drop from
without RV strain
or elevated biomarkers baseline or any with RV strain

Heparin Heparin Heparin


anticoagulation2 anticoagulation2 anticoagulation2

In clinic or emergency, Assess for Assess for


consider outpatient treatment3 contraindications to lysis4 contraindications to lysis4

Absolute Relative None None or


Absolute
Relative

Evidence of shock or
respiratory failure
and >2-hour delay to catheter
Watch and wait directed thrombolysis anticipated:
Consider IVC Alteplase
Large clot burden on CTPA1 100 mg over 2 h IV
filter and
Yes or tenecteplase
episodic hypotension tiered dose bolus
(SBP <90 mm Hg) per box top
or
HR/SBP consistently >1.0
or
Persistent hypotension,
SaO2 <92% with distress
hypoxemia with
No proximal clot or clot in
transit
Pharmacomechanical
catheter treatment
Consider open
thrombectomy
Significant delay anticipated

Alteplase 50 mg over 2 h IV, or


25 mg tenecteplase

Fig. 78.11.  Comprehensive treatment algorithm for diagnosed acute pulmonary embolism in a large,
full-service hospital. *Denotes a controversial pathway that is not available at many smaller hospitals.
Many experts believe that anticoagulation alone provides equivalent outcomes. 1, CTPA, CT pulmonary
angiography findings: filling defects in a lobar or more proximal artery; right ventricle (RV) > left ventricle
(LV) on CT scan; reflux of contrast into inferior vena cava (IVC) and liver. Abnormal echographic findings
include dilated or hypokinetic RV and estimated RV systolic pressure > 40 mm Hg.84,85 Elevated biomarkers
include brain natriuretic peptide (BNP) level > 90 pg/mL pro-BNP level > 900 pg/mL, or any troponin
concentration > 99th percentile for normal, with <10% coefficient of variability (ie, borderline or
higher).86-88 2, Unfractionated heparin 80 U/kg and then 16–18 U/kg/h to maintain PTT of 2–2.5. 3, See
Beam and colleagues.26 4, Contraindications to fibrinolysis. Absolute contraindications: 1, gastrointestinal
bleeding within previous 30 days; 2, active hemorrhage in any of the following sites at the time of
enrollment—intraperitoneal, retroperitoneal, pulmonary, uterine, bladder, or nose; 3, head trauma causing
loss of consciousness within previous 7 days; 4, any history of hemorrhagic stroke; 5, ischemic stroke
within the past year; 6, history of intraocular hemorrhage; 7, known or suspected intracranial metastasis;
8, liver failure with prothrombin time abnormal (international normalized ratio [INR] > 1.7); 9, surgery that
required opening of the chest cavity, peritoneum, skull, or spinal canal within the previous 14 days; 10,
subacute bacterial endocarditis under treatment; 11, pregnancy; 12, large pericardial effusion. Relative
contraindications: age > 75 years; dementia; surgery more than 30 days but less than 60 days prior; any
prior stroke; symptoms suggesting transient ischemic attack in the past 30 days; any prior gastrointestinal
bleeding; concurrent use of a thienopyridine (eg., clopidogrel); INR > 1.7 from warfarin use; any metastatic
cancer, tongue bites, recent fracture, recent fall with head strike, history of hematuria, nosebleeds, recent
dental extraction, or orthopedic surgery. HR, Heart rate; SBP, systolic blood pressure.

Descargado para Constanza Vicencio (constanza.vicencio@ua.cl) en Univ Antofagasta de ClinicalKey.es por Elsevier en septiembre 13, 2018.
Para uso personal exclusivamente. No se permiten otros usos sin autorización. Copyright ©2018. Elsevier Inc. Todos los derechos reservados.
C H APTER 78  Pulmonary Embolism and Deep Vein Thrombosis 1063

Standard Anticoagulation appropriate consultant should be contacted for urgent placement


of an inferior vena cava filter. If vena caval interruption cannot be
Patients with a high PTP, no contraindication to anticoagulation, performed within 12 hours, one option is to perform a baseline
and evidence of hemodynamic instability, including recent head CT scan, start an unfractionated heparin infusion at 18 U/
syncope, any hypotension, hypoxemia, or clinical evidence of right kg/hr (without a bolus), and admit the patient to the intensive
heart strain (criteria defined in Table 78.4 as more severe moder- care unit for close neurologic monitoring and frequent partial
ate PE or high-risk PE) should receive empirical heparin prior to thromboplastin time (PTT) determinations. The rationale for
waiting for the results of pulmonary vascular imaging. Patients using unfractionated heparin is that it can be reversed more reli-
with a positive imaging for DVT or PE should receive anticoagula- ably by discontinuing the heparin drip and administering prot-
tion using one of the agents in Table 78.3, administered in the ED amine, 1 mg/kg IV, than fractionated heparin. Case reports and
as soon as the diagnosis is confirmed.66 Low-molecular-weight series have suggested that inhaled nitric oxide might be helpful
(LMW) heparin is advantageous when compared to unfraction- for patients with severe PE and an absolute contraindication to
ated heparin based on robust meta-analyses that have clearly dem- anticoagulation, but this treatment has not yet been subjected to
onstrated lower rates of major hemorrhage, heparin-induced rigorous study.77
thrombocytopenia, and VTE, with similar cost.67 Patients can now Most patients with PE state that they feel better the day after
be anticoagulated in the ED with apixaban (Eliquis) or rivaroxa- starting heparin anticoagulation, and more than half go on to
ban (Xarelto), which are orally available agents that specifically nearly a full recovery of pre-PE health status. The in-hospital
inhibit one enzyme in the clotting pathway. These drugs can be mortality rate of patients diagnosed with PE who remain hemo-
started without prior or concomitant use of heparin, and they dynamically stable while in the ED was thought to be 10%, but a
provide therapeutic anticoagulation effect as rapidly as subcuta- recent large, multicenter, US-based registry of 1880 patients diag-
neous LMW heparin (see Table 78.3). By obviating the need for nosed with PE in the ED found the in-hospital mortality rate
twice-daily subcutaneous injections and blood monitoring, these directly attributable to PE as 1.1% and an all-cause mortality rate
drugs can facilitate outpatient treatment of DVT and PE. of 5.4%.29 Approximately 10% to 20% of PE survivors complain
Patients with a history of heparin-induced thrombocytopenia of persistent dyspnea and exercise intolerance that permanently
should receive fondaparinux, argatroban, apixaban, or rivaroxa- degrades their quality of life.78
ban. Most hematologists, internists, and obstetricians prefer that
pregnant patients with VTE receive twice-daily LMW heparin.68 Fibrinolytic (Thrombolytic) Therapy
The anticoagulant effect of unfractionated heparin can be
almost completely and rapidly reversed with protamine, whereas Fibrinolytic therapy in PE remains a controversial treatment
LMW heparin can only be 50% neutralized with protamine. Prot- option. Recent meta-analyses of randomized trials that compared
amine has no effect on fondaparinux, rivaroxaban, or apixaban. fibrinolysis plus heparin to heparin alone have reached different
At present, based on data in healthy volunteers, the best agent to conclusions about mortality benefit, with one study finding sig-
correct coagulopathy from apixaban or rivaroxaban is four-factor nificant improvement79 and another no difference80 in mortality.
activated prothrombin complex (Beriplex P/N or K-Centra, 50 U/ Most experts, even those generally opposed to fibrinolysis, believe
kg, IV).69,70 No clinical trials have been published to test the effect that patients with arterial hypotension (systolic blood pressure <
of these agents on bleeding in people with apixaban or rivaroxa- 90 mm Hg or >40-mm Hg drop from baseline) should receive
ban coagulopathy. full-dose systemic fibrinolysis (100 mg of alteplase over 2 hours
Regarding isolated subsegmental PE, if the patient has no evi- or tiered-dose tenecteplase, per the TNKase label). No one doubts
dence of DVT on bilateral lower extremity ultrasonography, no that the bleeding risk increases with systemic fibrinolysis, but the
signs of cardiopulmonary stress (eg, normal biomarkers, normal risk of intracranial hemorrhage appears to be mostly confined to
ECG), and no ongoing major risk for thrombosis (eg, active patients older than 65 years.79 The “to lyse or not to lyse PE” con-
malignancy, atrial fibrillation), it is reasonable and prudent to troversy has been made more complex by recent studies suggest-
withhold anticoagulation for patients with isolated subsegmental ing a possible lower risk of significant hemorrhage associated with
filling defects on CTPA.71 If a patient with a negative CTPA scan the lower, half-dose alteplase (50 mg over 2 hours), administered
has ongoing dyspnea and signs of pulmonary hypertension— by peripheral vein.81 Moreover, many large treatment centers have
enlarged right ventricle,72 enlarged pulmonary artery, or reflux of adopted the use of catheter-directed thrombolysis, which admin-
contrast into the liver, the mosaic pattern, acute pulmonary isters the fibrinolytic directly into the thrombus, with or without
hypertension on the ECG73—or hypoxemia without an apparent adjunctive ultrasonic energy.82 The potential advantage of this
alternative cause, at minimum, the patient should have transtho- approach is a lower risk of hemorrhage due to the lower dose of
racic echocardiography performed. If this demonstrates pulmo- fibrinolytic agent (eg, 20–25 mg of alteplase infused intrathrom-
nary hypertension or right ventricular overload, the patient bus over 24 hours.) No evidence has yet demonstrated a survival
should be referred or admitted to a pulmonary specialist to guide advantage or any patient-oriented advantage of catheter-directed
further testing.40,41 therapy. However, one small randomized trial has demonstrated
Recent evidence has indicated that up to 50% of outpatients improved quality of life end points with bolus administration of
diagnosed with PE may be stable enough to be treated as outpa- tenecteplase for severe submassive PE.83
tients (low-risk criteria; see Table 78.4).74,75 In settings where good The clinical course of patients with obstructive PE can be
follow-up can be obtained, and the patient can be taught to self- unpredictable. Many patients with massive PE remain stable in
administer LMW heparin and can access an anticoagulation clinic the ED. Other patients are stable on arrival, but progressively
within 48 hours, a low-risk patient with PE can be discharged deteriorate over hours as right ventricular function declines. Of
from the ED. My choice, however, is to implement a protocol that ED patients without hypotension, 3% experience cardiac arrest
includes the Hestia criteria to select low-risk patients (see Box while in the ED and die within 24 hours.33 A patient can be stable
78.1), together with monotherapy with apixaban or rivaroxaban; and then hypotensive within minutes because of the highly vari-
this has been associated with low rates of complications and eco- able effect of the clot on right ventricular outflow obstruction,
nomic advantages.26,76 especially when it straddles the main pulmonary artery (Fig.
For a patient diagnosed with PE in the presence of a major 78.12). Additional mechanisms of rapid instability include new
contraindication to anticoagulation, such as a recent cerebral embolization of clot material, release of mediators of pulmonary
hemorrhage or large cerebral infarction, or brain metastases, the vasospasm, sudden bradyasystolic arrhythmias, and respiratory

Descargado para Constanza Vicencio (constanza.vicencio@ua.cl) en Univ Antofagasta de ClinicalKey.es por Elsevier en septiembre 13, 2018.
Para uso personal exclusivamente. No se permiten otros usos sin autorización. Copyright ©2018. Elsevier Inc. Todos los derechos reservados.
1064 PART III  Medicine and Surgery  |  SECTION Four  Vascular System

narrow-complex tachycardia to an incomplete right bundle


branch block to a complete right bundle branch block (Fig. 78.13)
is evidence of life-threatening pulmonary hypertension and
impending cardiac arrest.
Clinical evidence of impending or actual respiratory failure
indicates the need for prompt endotracheal intubation using a
standard rapid sequence intubation technique, preferably with
ketamine or etomidate for induction of anesthesia with neuro-
muscular blockade. Other induction agents that depress cardiac
function or reduce preload may precipitate severe hypotension
and should be avoided or their dosage reduced. The effect
of biphasic, positive pressure–assisted noninvasive ventilation
(BiPAP) on hemodynamics with massive PE has not been studied.
For patients with PE and persistent hypotension, the role of
A volume loading to resuscitate massive PE remains uncertain.
Most experts use norepinephrine as the vasopressor of choice to
attempt to increase blood pressure. In the case of impending
respiratory or cardiac arrest, fibrinolytic therapy should be
strongly considered.

Surgical Embolectomy
For patients with known floating thrombi in the right heart or
patients with severe refractory hypotension, surgery is the most
likely intervention to save the patient’s life. Surgical embolectomy
commonly includes extracorporeal cardiopulmonary bypass and
an experienced cardiothoracic surgeon. Surgical embolectomy
may be the best option for patients who have severe PE with a
contraindication to fibrinolysis; however, extracorporeal perfu-
sion requires intensive heparin anticoagulation, and the patient’s
mental status cannot be monitored during surgery—a key concern
in patients with a high risk of intracranial hemorrhage.
B Numerous case reports have suggested heroic results from the
bolus administration of thrombolytic therapy to patients with
Fig. 78.12.  CT evidence suggesting more severe PE. A, Proximal pul- cardiac arrest from PE. The administration of fibrinolytic therapy
monary embolism on a contrast-enhanced CT scan of the chest. This CT does not absolutely preclude surgical intervention. Patients who
scan is at the level of the bifurcation of the main pulmonary artery. The
left main branch of the pulmonary artery shows a massive filling defect
have been treated with a fibrinolytic agent can undergo sternot-
(arrows). B, Evidence of right ventricular strain, shown by the larger size omy or thoracotomy for embolectomy and survive without fatal
of the right ventricle compared with the left ventricle. hemorrhage. The decision to perform embolectomy ultimately
resides with the cardiac surgeon.

Disposition
failure. Clues to oncoming cardiopulmonary decompensation Table 78.5 summarizes the criteria that can be used to risk-stratify
include worsening respiratory distress and worsening hypoxemia, patients with PE into four groups. This stratification may help
rising shock index (the heart rate divided by the systolic blood guide the decision to place the patient in an intensive care
pressure), systolic arterial blood pressure less than 90 mm Hg, unit versus an intermediate or regular inpatient bed and whether
and syncope or a sharp change in mental status, including seizure- to administer heparin only or consider escalated therapy (see
like convulsive episodes. Deterioration in the ECG from a Fig. 78.11).

Descargado para Constanza Vicencio (constanza.vicencio@ua.cl) en Univ Antofagasta de ClinicalKey.es por Elsevier en septiembre 13, 2018.
Para uso personal exclusivamente. No se permiten otros usos sin autorización. Copyright ©2018. Elsevier Inc. Todos los derechos reservados.
C H APTER 78  Pulmonary Embolism and Deep Vein Thrombosis 1065

I aVR V1 V4

II aVL V2 V5

III aVF V3 V6
A

I aVR V1 V4

II aVL V2 V5

III aVF V3 V6
B
Fig. 78.13.  Serial electrocardiograms obtained 2 minutes apart show the progression from a narrow
complex rhythm (A) to a right bundle branch block pattern (B) in a patient with massive bilateral pul-
monary emboli. Shortly after the second tracing was obtained, the patient developed cardiovascular
collapse refractory to vigorous resuscitation efforts.

TABLE 78.5 

Risk Stratification and Associated Treatment Recommendations for Acute PE


CATEGORY CRITERIA ACTION
89
Low-risk PE sPESI of 0 • Begin anticoagulant treatment
Hestia criteria negative (see Box 78.1)90 (see Table 78.3)
SBP > 90 mm Hg at all times and all of the following: no proximal clot or RV • Optional admission to unmonitored
dilation on CTPA, shock index < 1, SaO2 > 94%, no pulmonary hypertension regular bed
on ECG (Daniel score < 3), normal troponin, BNP, or pro-BNP level • Consider outpatient treatment if adequate
• Compliance and follow-up can be assured
Moderate-risk PE SBP > 90 mm Hg at all times and any one of the following: • Begin anticoagulant treatment
Proximal clot and RV > LV on CTPA scan (see Fig. 78.12)72,84,85,88 (see Table 78.3)
Elevated troponin or BNP level (>90 pg/mL) or pro-BNP level (>900 pg/mL) • Fibrinolytics in minority of cases
Echocardiogram with any degree of right ventricular hypokinesis • Admission to a telemetry bed
More severe Any moderate risk criteria and appearance of at respiratory distress • Begin heparin treatment
(submassive); Shock index > 1 and severe right ventricular hypokinesis on echocardiography • Fibrinolytic treatment in most patients
moderate-risk PE Worsening Daniel score, particularly a new incomplete right bundle branch without contraindications in the ED
block (RBBB) or progression of incomplete RBBB to complete RBBB • Admission to a step-down or intensive
SaO2 < 90% and serum troponin level clearly elevated care unit
New altered mental status
High-risk (major) PE Any SBP <90 mm Hg or <20 mm Hg below documented baseline and • Begin heparin treatment
appearance of distress • Fibrinolytic treatment in all patients
Any persistent SBP <90 mm Hg, regardless of appearance without contraindications in the ED
• Admission to intensive care unit
BNP, Brain natriuretic peptide; CTPA, computed tomography pulmonary angiography; ED, emergency department; PE, pulmonary embolism; RBBB, right bundle branch block;
RV/LV, right ventricle−left ventricle ratio; SBP, systolic blood pressure.

Descargado para Constanza Vicencio (constanza.vicencio@ua.cl) en Univ Antofagasta de ClinicalKey.es por Elsevier en septiembre 13, 2018.
Para uso personal exclusivamente. No se permiten otros usos sin autorización. Copyright ©2018. Elsevier Inc. Todos los derechos reservados.
1066 PART III  Medicine and Surgery  |  SECTION Four  Vascular System

KEY CONCEPTS
• Deep vein thrombosis often presents as a nonspecific crampy • When duplex ultrasound is not available, patients with a moderate to
sensation in the upper or lower extremity without obvious swelling. high PTP for DVT should receive empirical LMW heparin, oral
• An ELISA or immunoturbidimetric D-dimer concentration less than apixaban, or rivaroxaban while awaiting the availability of ultrasound
500 ng/mL can exclude DVT in patients with a low pretest imaging; patients with a low PTP, or moderate to high PTP with a
probability (PTP). negative D-dimer, do not need empirical anticoagulation while they
• A negative three-point ultrasound, together with a negative wait for diagnostic imaging.
quantitative D-dimer test result, excludes DVT with any PTP. • Patients at a low or moderate PTP for PE should have a D-dimer test
• A negative three-point ultrasound in a patient with a moderate or done prior to performing pulmonary vascular imaging; PE can be
high PTP for DVT should have additional testing, including a D-dimer ruled out in non–high PTP, with a D-dimer adjusted for age
test or repeat venous ultrasound within 3 to 7 days. according to this formula: age × 10 ng/mL.
• An enzyme-linked immunosorbent assay (ELISA) or • A patient with a PTP less than 2% need not be tested for PE.
immunoturbidimetric D-dimer concentration less than 500 ng/mL can • Patients with PE and low risk according to the Hestia criteria (see
rule out PE non–high PTP patients, including all pregnant patients. Box 78.1) can be treated at home, provided they have adequate
• A patient at a low PTP may have the diagnosis of DVT effectively follow-up.
excluded by a negative three-point venous duplex ultrasound • Patients with PE and arterial hypotension (systolic blood pressure <
performed by a qualified emergency clinician or radiologist. 90 mm Hg) should receive systemic fibrinolysis unless they have a
• A single, whole-leg ultrasound excludes DVT in all pretest contraindication to fibrinolysis.
probabilities.

The references for this chapter can be found online by accessing the accompanying Expert Consult website.

Descargado para Constanza Vicencio (constanza.vicencio@ua.cl) en Univ Antofagasta de ClinicalKey.es por Elsevier en septiembre 13, 2018.
Para uso personal exclusivamente. No se permiten otros usos sin autorización. Copyright ©2018. Elsevier Inc. Todos los derechos reservados.
C H APTER 78  Pulmonary Embolism and Deep Vein Thrombosis 1066.e1

REFERENCES
1. Kline JA, Kabrhel C: Emergency Evaluation for Pulmonary Embolism, Part 1: Clinical 31. Kline JA, Corredor DM, Hogg MM, et al: Normalization of vital signs does not reduce
Factors that Increase Risk. J Emerg Med 2015. the probability of acute pulmonary embolism in symptomatic emergency depart-
2. Decousus H, Prandoni P, Mismetti P, et al: Fondaparinux for the treatment of ment patients. Acad Emerg Med 19:11–17, 2012.
superficial-vein thrombosis in the legs. N Engl J Med 363:1222–1232, 2010. 32. Dresden S, Mitchell P, Rahimi L, et al: Right ventricular dilatation on bedside echo-
3. Sermsathanasawadi N, Suparatchatpun P, Pumpuang T, et al: Comparison of clinical cardiography performed by emergency physicians aids in the diagnosis of pulmonary
prediction scores for the diagnosis of deep vein thrombosis in unselected population embolism. Ann Emerg Med 63:16–24, 2014.
of outpatients and inpatients. Phlebology 30:469–474, 2015. 33. Lin BW, Schreiber DH, Liu G, et al: Therapy and outcomes in massive pulmonary
4. Le ME, Genty C, Meunier J, et al: Validation of the LEFt score, a newly proposed embolism from the Emergency Medicine Pulmonary Embolism in the Real World
diagnostic tool for deep vein thrombosis in pregnant women. Thromb Res 134:664– Registry. Am J Emerg Med 30:1774–1781, 2012.
667, 2014. 34. Logan JK, Pantle H, Huiras P, et al: Evidence-based diagnosis and thrombolytic treat-
5. Vinson DR, Patel JP, Irving CS: Pretest probability estimation in the evaluation of ment of cardiac arrest or periarrest due to suspected pulmonary embolism. Am J
patients with possible deep vein thrombosis. Am J Emerg Med 29:594–600, 2011. Emerg Med 32:789–796, 2014.
6. Wang B, Lin Y, Pan FS, et al: Comparison of empirical estimate of clinical pretest 35. Kline JA, Neumann D, Haug MA, et al: Decreased facial expression variability in
probability with the Wells score for diagnosis of deep vein thrombosis. Blood Coagul patients with serious cardiopulmonary disease in the emergency care setting. Emerg
Fibrinolysis 24:76–81, 2013. Med J 32:3–8, 2014.
7. Mortazavi M, Samiee MM, Spencer FA: Incidence of deep vein thrombosis in erysip- 36. Smith SB, Geske JB, Maguire JM, et al: Early Anticoagulation is associated with
elas or cellulitis of the lower extremities. Int J Dermatol 52:279–285, 2013. reduced mortality for acute pulmonary embolism. Chest 137:1382–1389, 2010.
8. Gunderson CG, Chang JJ: Overuse of compression ultrasound for patients with lower 37. Torres-Macho J, Mancebo-Plaza AB, Crespo-Gimenez A, et al: Clinical features of
extremity cellulitis. Thromb Res 134:846–850, 2014. patients inappropriately undiagnosed of pulmonary embolism. Am J Emerg Med
9. Gunderson CG, Chang JJ: Risk of deep vein thrombosis in patients with cellulitis and 31:1646–1650, 2013.
erysipelas: a systematic review and meta-analysis. Thromb Res 132:336–340, 2013. 38. Kline JA, Slattery D, O’Neil BJ, et al: Clinical fFeatures of patients with pulmonary
10. Kabrhel C, Mark Courtney D, Camargo CA, Jr, et al: Factors associated with positive embolism and a negative PERC rule result. Ann Emerg Med 61:122–124, 2013.
D-dimer results in patients evaluated for pulmonary embolism. Acad Emerg Med 39. Kline JA, Hogg MM, Courtney DM, et al: D-dimer threshold increase with pretest
17:589–597, 2010. probability unlikely for pulmonary embolism to decrease unnecessary computerized
11. Johnson SA, Stevens SM, Woller SC, et al: Risk of deep vein thrombosis following a tomographic pulmonary angiography. J Thromb Haemost 10:572–581, 2012.
single negative whole-leg compression ultrasound: a systematic review and meta- 40. Kline JA, Russell FM, Lahm T, et al: Derivation of a screening tool to identify patients
analysis. JAMA 303:438–445, 2010. with right ventricular dysfunction or tricuspid regurgitation after negative computer-
12. Linkins LA, Bates SM, Lang E, et al: Selective D-dimer testing for diagnosis of a first ized tomographic pulmonary angiography of the chest. Pulm Circ 5:171–183, 2015.
suspected episode of deep venous thrombosis: a randomized trial. Ann Intern Med 41. Russell FM, Moore CL, Courtney DM, et al: Independent evaluation of a simple
158:93–100, 2013. clinical prediction rule to identify right ventricular dysfunction in patients with
13. Go MR, Kiser D, Wald P, et al: Clinical evaluation of suspected deep vein thrombosis shortness of breath. Am J Emerg Med 33:542–547, 2015.
guides the decision to anticoagulate prophylactically but does not impact the decision 42. Feng LB, Pines JM, Yusuf HR, et al: trends in computed tomography use and diag-
to perform afte-hours duplex venous scanning or increase its yield. J Vasc Surg noses in emergency department visits by patients with symptoms suggestive of pul-
57:1597–1602, 2013. monary embolism, 2001-2009. Acad Emerg Med 20:1033–1040, 2013.
14. Pomero F, Dentali F, Borretta V, et al: Accuracy of emergency physician-performed 43. Schissler AJ, Rozenshtein A, Kulon ME, et al: CT pulmonary angiography: increas-
ultrasonography in the diagnosis of deep-vein thrombosis: a systematic review and ingly diagnosing less severe pulmonary emboli. PLoS ONE 8:e65669, 2013.
meta-analysis. Thromb Haemost 109:137–145, 2013. 44. Adams DM, Stevens SM, Woller SC, et al: Adherence to PIOPED II investigators’
15. Pacouret G, Alison D, Pottier JM, et al: Free-floating thrombus and embolic risk in recommendations for computed tomography pulmonary angiography. Am J Med
patients with angiographically confirmed proximal deep venous thrombosis. A pro- 126:36–42, 2013.
spective study. Arch Intern Med 157:305–308, 1997. 45. Lucassen W, Geersing GJ, Erkens PM, et al: Clinical decision rules for excluding
16. Kahn SR, Shapiro S, Wells PS, et al: Compression stockings to prevent post- pulmonary embolism: a meta-analysis. Ann Intern Med 155:448–460, 2011.
thrombotic syndrome: a randomised placebo-controlled trial. Lancet 383:880–888, 46. Lessler AL, Isserman JA, Agarwal R: Pet al: Testing low-risk patients for suspected
2014. pulmonary embolism: a decision analysis. Ann Emerg Med 55:316–326, 2010.
17. Decousus H, Prandoni P, Mismetti P, et al: Fondaparinux for the treatment of 47. Siccama RN, Janssen KJ, Verheijden NA, et al: Systematic review: diagnostic accuracy
superficial-vein thrombosis in the legs. N Engl J Med 363:1222–1232, 2010. of clinical decision rules for venous thromboembolism in elderly. Ageing Res Rev
18. Schwarz T, Buschmann L, Beyer J, et al: Therapy of isolated calf muscle vein throm- 10:304–313, 2011.
bosis: a randomized, controlled study. J Vasc Surg 52:1246–1250, 2010. 48. Singh B, Mommer SK, Erwin PJ, et al: Pulmonary embolism rule-out criteria (PERC)
19. De Martino RR, Wallaert JB, Rossi AP, et al: A meta-analysis of anticoagulation for in pulmonary embolism—revisited: a systematic review and meta-analysis. Emerg
calf deep venous thrombosis. J Vasc Surg 56:228–237, 2012. Med J 30:701–706, 2013.
20. Mino JS, Gutnick JR, Monteiro R, et al: Line-associated thrombosis as the major 49. Kline JA, Marchick MR, Kabrhel C, et al: Prospective study of the frequency and
cause of hospital-acquired deep vein thromboses: an analysis from National Surgical outcomes of patients with suspected pulmonary embolism administered heparin
Quality Improvement Program data and a call to reassess prophylaxis strategies. Am prior to confirmatory imaging. Thromb Res 129:e25–e28, 2012.
J Surg 208:45–49, 2014. 50. Schouten HJ, Geersing GJ, Koek HL, et al: Diagnostic accuracy of conventional or
21. Chopra V, Anand S, Hickner A, et al: Risk of venous thromboembolism associated age adjusted D-dimer cut-off values in older patients with suspected venous throm-
with peripherally inserted central catheters: a systematic review and meta-analysis. boembolism: systematic review and meta-analysis. BMJ 346:f2492, 2013.
Lancet 382:311–325, 2013. 51. Righini M, van Es J, den Exter PL, et al: Age-adjusted D-dimer cutoff levels to rule
22. Di NM, Van Sluis GL, Bossuyt PM, et al: Accuracy of diagnostic tests for clinically out pulmonary embolism: the ADJUST-PE study. JAMA 311:1117–1124, 2014.
suspected upper extremity deep vein thrombosis: a systematic review. J Thromb 52. Mos IC, Klok FA, Kroft LJ, et al: Safety of ruling out acute pulmonary embolism by
Haemost 8:684–692, 2010. normal computed tomography pulmonary angiography in patients with an indica-
23. Kearon C, Akl EA, Comerota AJ, et al: American College of Chest Physicians: Anti- tion for computed tomography: systematic review and meta-analysis. J Thromb
thrombotic therapy for VTE disease: Antithrombotic Therapy and Prevention of Haemost 7:1491–1498, 2009.
Thrombosis, 9th ed: American College of Chest Physicians Evidence-Based Clinical 53. Courtney DM, Miller CD, Smithline HA, et al: Prospective multi-center assessment
Practice Guidelines. Chest 141(Suppl):e419S–e494S, 2012. of interobserver agreement for radiologist interpretation of multidetector CT angi-
24. Lee JA, Zierler BK, Zierler RE: The risk factors and clinical outcomes of upper extrem- ography for pulmonary embolism. J Thromb Haemost 8:533–540, 2010.
ity deep vein thrombosis. Vasc Endovascular Surg 46:139–144, 2012. 54. Burnside PR, Green E, Kline JA: Indirect computed tomography venography: a report
25. Kahn SR: The post thrombotic syndrome. Thromb Res 127(Suppl 3):S89–S92, 2011. of vascular opacification. Emerg Radiol 17:195–201, 2010.
26. Beam DM, Kahler ZP, Kline JA: Immediate discharge and home treatment of low-risk 55. Meng K, Hu X, Peng X, et al: Incidence of venous thromboembolism during preg-
venous thromboembolism diagnosed in two U.S. emergency departments with riva- nancy and the puerperium: a systematic review and meta-analysis. J Matern Fetal
roxaban: a one-year preplanned analysis. Acad Emerg Med 22:788–795, 2015. Neonatal Med 28:245–253, 2015.
27. Courtney DM, Kline JA, Kabrhel C, et al: Clinical features from the history and 56. Kline JA, Richardson DM, Than MP, et al: Systematic review and meta-analysis of
physical examination that predict the presence or absence of pulmonary embolism pregnant patients investigated for suspected pulmonary embolism in the emergency
in symptomatic emergency department patients: results of a prospective, multicenter department. Acad Emerg Med 21:949–959, 2014.
study. Ann Emerg Med 55:305–315, 2010. 57. Leung AN, Bull TM, Jaeschke R, et al: An official American Thoracic Society/Society
28. De S, V, Rossi E: Testing for inherited thrombophilia and consequences for anti- of Thoracic Radiology clinical practice guideline: evaluation of suspected pulmonary
thrombotic prophylaxis in patients with venous thromboembolism and their rela- embolism in pregnancy. Am J Respir Crit Care Med 184:1200–1208, 2011.
tives. A review of the Guidelines from Scientific Societies and Working Groups. 58. Tan M, Huisman MV: The diagnostic management of acute venous thromboembo-
Thromb Haemost 110:697–705, 2013. lism during pregnancy: recent advancements and unresolved issues. Thromb Res
29. Pollack CV, Schreiber D, Goldhaber SZ, et al: Clinical characteristics, management, 127(Suppl 3):S13–S16, 2011.
and outcomes of patients diagnosed with acute pulmonary embolism in the emer- 59. Kline JA, Hambleton GW, Hernandez J: D-dimer concentrations in normal preg-
gency department: initial report of EMPEROR (Multicenter Emergency Medicine nancy: new diagnostic thresholds are needed. Clin Chem 51:825–829, 2005.
Pulmonary Embolism in the Real World Registry). J Am Coll Cardiol 57:700–706, 60. Chan WS, Lee A, Spencer FA, et al: D-dimer testing in pregnant patients: towards
2011. determining the next “level” in the diagnosis of deep vein thrombosis. J Thromb
30. Beam DM, Neto-Neves EM, Stubblefield WB, et al: Anesthetized swine model of Haemost 8:1004–1011, 2010.
acute pulmonary embolism producing right ventricular dysfunction: comparison of 61. Chan WS, Ginsberg JS: Management of venous thromboembolism in pregnancy.
isoflurane and alpha chloralose. Comp Med 65:54–61, 2014. In van Beek EJR, Buller HR, Oudkerk M, editors: Deep vein thrombosis and

Descargado para Constanza Vicencio (constanza.vicencio@ua.cl) en Univ Antofagasta de ClinicalKey.es por Elsevier en septiembre 13, 2018.
Para uso personal exclusivamente. No se permiten otros usos sin autorización. Copyright ©2018. Elsevier Inc. Todos los derechos reservados.
1066.e2 PART III  Medicine and Surgery  |  SECTION Four  Vascular System

pulmonary embolism, Chichester, West Sussex, England, 2009, John Wiley & Sons, 77. Kline JA, Hernandez-Nino J, Garrett JS, et al: Pilot study of a protocol to administer
pp 353–371. inhaled nitric oxide to treat severe acute submassive pulmonary embolism. Emerg
62. Nijkeuter M, Tan M, Middeldorp S, et al: Safety of ruling out pulmonary embolism Med J 31:459–463, 2014.
(PE) in pregnancy by computed tomography pulmonary angiography (CTPA). 78. van Es J, den Exter PL, Kaptein AA, et al: Quality of life after pulmonary embolism
J Thromb Haemost 11(Suppl 2):2013. as assessed with SF-36 and PEmb-QoL. Thromb Res 132:500–505, 2013.
63. Shahir K, Goodman LR, Tali A, et al: Pulmonary embolism in pregnancy: CT pul- 79. Chatterjee S, Chakraborty A, Weinberg I, et al: Thrombolysis for pulmonary embo-
monary angiography versus perfusion scanning. AJR Am J Roentgenol 195:W214– lism and risk of all-cause mortality, major bleeding, and intracranial hemorrhage: a
W220, 2010. meta-analysis. JAMA 311:2414–2421, 2014.
64. Chatterson LC, Leswick DA, Fladeland DA, et al: Lead versus bismuth-antimony 80. Nakamura S, Takano H, Kubota Y, et al: Impact of the efficacy of thrombolytic
shield for fetal dose reduction at different gestational ages at CT pulmonary angiog- therapy on the mortality of patients with acute submassive pulmonary embolism: a
raphy. Radiology 260:560–567, 2011. meta-analysis. J Thromb Haemost 12:1086–1095, 2014.
65. Provias T, Dudzinski DM, Jaff MR, et al: The Massachusetts General Hospital Pul- 81. Zhang Z, Zhai ZG, Liang LR, et al: Lower dosage of recombinant tissue-type plas-
monary Embolism Response Team (MGH PERT): creation of a multidisciplinary minogen activator (rt-PA) in the treatment of acute pulmonary embolism: a system-
program to improve care of patients with massive and submassive pulmonary embo- atic review and meta-analysis. Thromb Res 133:357–363, 2014.
lism. Hosp Pract (1995) 42:31–37, 2014. 82. Kucher N, Boekstegers P, Muller OJ, et al: Randomized, controlled trial of ultrasound-
66. Castellucci LA, Cameron C, Le GG, et al: Clinical and safety outcomes associated with assisted catheter-directed thrombolysis for acute intermediate-risk pulmonary embo-
treatment of acute venous thromboembolism: a systematic review and meta-analysis. lism. Circulation 129:479–486, 2014.
JAMA 312:1122–1135, 2014. 83. Kline JA, Kabrhel C, Courtney DM, et al: Treatment of submassive pulmonary embo-
67. Bochenek T, Nizankowski R: The treatment of venous thromboembolism with lism with tenecteplase or placebo: cardiopulmonary outcomes at three months
low-molecular-weight heparins. A meta-analysis. Thromb Haemost 107:699–716, (TOPCOAT): Multicenter double-blind, placebo-controlled randomized trial.
2012. J Thromb Haemost 12:549–568, 2014.
68. Gandara E, Carrier M, Rodger MA: Management of pregnancy associated venous- 84. Jimenez D, Lobo JL, Monreal M, et al: Prognostic significance of multidetector CT
thromboembolism: a survey of practices. Thromb J 12:12, 2014. in normotensive patients with pulmonary embolism: results of the protect study.
69. Levi M, Moore KT, Castillejos CF, et al: Comparison of three-factor and four-factor Thorax 69:109–115, 2013.
prothrombin complex concentrates regarding reversal of the anticoagulant effects of 85. Trujillo-Santos J, den Exter PL, Gomez V, et al: Computed tomography-assessed right
rivaroxaban in healthy volunteers. J Thromb Haemost 12:1428–1436, 2014. ventricular dysfunction and risk stratification of patients with acute non-massive
70. Eerenberg ES, Kamphuisen PW, Sijpkens MK, et al: Reversal of rivaroxaban and pulmonary embolism: systematic review and meta-analysis. J Thromb Haemost
dabigatran by prothrombin complex concentrate: a randomized, placebo-controlled, 11:1823–1832, 2013.
crossover study in healthy subjects. Circulation 124:1573–1579, 2011. 86. Jaff MR, McMurtry MS, Archer SL, et al: Management of massive and submassive
71. Carrier M, Righini M, Le GG: Symptomatic subsegmental pulmonary embolism: pulmonary embolism, iliofemoral deep vein thrombosis, and chronic thromboem-
what is the next step? J Thromb Haemost 10:1486–1490, 2012. bolic pulmonary hypertension: a scientific statement from the American Heart Asso-
72. Becattini C, Agnelli G, Germini F, et al: Computed tomography to assess risk of death ciation. Circulation 1:1788–1830, 2011.
in acute pulmonary embolism: a meta-analysis. Eur Respir J 43:1678–1690, 2014. 87. Aviram G, Rogowski O, Gotler Y, et al: Real time risk stratification of patients with
73. Shopp JD, Stewart LK, Emmett TW, et al: Findings from 12-lead electrocardiography acute pulmonary embolism by grading the reflux of contrast into the inferior vena
that predict circulatory shock from pulmonary embolism: systematic review and cava on computerized tomographic pulmonary angiography. J Thromb Haemost
meta-analysis. Acad Emerg Med 22:1127–1137, 2015. 6:1488–1493, 2008.
74. Vinson DR, Zehtabchi S, Yealy DM: Can selected patients with newly diagnosed 88. Vedovati MC, Becattini C, Agnelli G, et al: Multidetector CT scan for acute pulmo-
pulmonary embolism be safely treated without hospitalization? A systematic review. nary embolism: embolic burden and clinical outcome. Chest 142:1417–1424, 2012.
Ann Emerg Med 60:651–662, 2012. 89. Jimenez D, Kopecna D, Tapson V, et al: Derivation and validation of multimarker
75. Piran S, Le GG, Wells PS, et al: Outpatient treatment of symptomatic pulmonary prognostication for normotensive patients with acute symptomatic pulmonary
embolism: a systematic review and meta-analysis. Thromb Res 132:515–519, 2013. embolism. Am J Respir Crit Care Med 189:718–726, 2014.
76. Kahler ZP, Beam DM, Kline JA: Cost of treating venous thromboembolism with 90. Zondag W, den Exter PL, Crobach MJ, et al: Comparison of two methods for selection
heparin and warfarin versus home treatment with rivaroxaban. Acad Emerg Med of out of hospital treatment in patients with acute pulmonary embolism. Thromb
22:796–802, 2015. Haemost 109:47–52, 2013.

CHAPTER 78: QUESTIONS & ANSWERS


78.1. Which of the following statements concerning the 78.3. A 29-year-old woman presents with onset of left calf pain
D-dimer protein level is most true? and mild swelling over a 24-hour period. She is 26 weeks
A. It is derived from the enzymatic breakdown of pregnant, with no other medical problems and no other
thrombin. symptoms. The D-dimer level is 845 ng/mL. Lower
B. It is elevated in patients with acute febrile illness. extremity duplex ultrasonography is negative. What would
C. The concentration is higher in smokers but unaffected be the most appropriate course of action?
by advancing age. A. Contrast CT scanning of the chest
D. The concentration is proportional to the size of the B. Empirical anticoagulation
clot. C. Repeat ultrasound in 2 or 3 days
E. The serum half-life is 48 to 60 hours, making it D. Repeat ultrasound in 24 hours
sensitive for older clots.   scan of the chest
E. V/Q
Answer: D. The D-dimer concentration varies directly with clot Answer: C. In moderate- to high-risk patients with an elevated
burden. It is derived from the enzymatic breakdown of fibrin, with D-dimer level, a single ultrasound may be insufficient. A repeat
a serum half-life of 8 hours, making it less sensitive for older, more study in 2 to 7 days is often sufficient to confirm the diagnosis.
mature clots. Levels are not higher in smokers but may be elevated The lack of pulmonary symptoms precludes the need for lung and
with advanced age and immobility. It is not elevated by febrile embolus evaluation at this time. During pregnancy, there is a
illness. progressive rise in baseline D-dimer concentration; thus, a normal
value is useful but an elevated level is of no discriminatory value.
78.2. What is the sensitivity of venous duplex ultrasonography
for detecting a proximal DVT? 78.4. A 43-year-old woman presents with acute pain and
A. 80% swelling of her right saphenous vein. Symptoms have
B. 85% occurred over 48 hours. She does not smoke and has no
C. 90% significant past medical history. Vital signs are
D. 95% unremarkable, and the physical examination is also
E. 100% unremarkable, except for isolated swelling, erythema,
tenderness, and increased firmness along the track of the
Answer: D. The sensitivity of a single scan is 95%. Thus, 5% are
right saphenous vein from the malleolus to 4 cm below
missed.

Descargado para Constanza Vicencio (constanza.vicencio@ua.cl) en Univ Antofagasta de ClinicalKey.es por Elsevier en septiembre 13, 2018.
Para uso personal exclusivamente. No se permiten otros usos sin autorización. Copyright ©2018. Elsevier Inc. Todos los derechos reservados.
C H APTER 78  Pulmonary Embolism and Deep Vein Thrombosis 1066.e3

the tibial plateau. There is no calf or thigh swelling or Answer: D. So-called effort thrombosis is often seen in healthy
tenderness. Which of the following would be appropriate patients after vigorous exercise. Many of these are later found to
management? have anatomic abnormalities relating to the subclavian and axil-
A. Nonsteroidal antiinflammatory drugs (NSAIDs) and lary vein. Approximately 50% of upper extremity DVTs are related
compression stockings to indwelling catheters. Catheter removal is not always mandatory.
B. Antistaphylococcal antibiotics and antiinflammatory Appropriate treatment for upper extremity DVT includes full
agents anticoagulation and is sometimes accompanied by fibrinolysis or
C. Antistaphylococcal antibiotics and elevation for 24 to thrombectomy. The incidence of pulmonary embolus is the same
48 hours as for femoral DVTs. Many patients remain symptomatic, with
D. Systemic anticoagulation ongoing arm pain and swelling, despite appropriate treatment.
E. Ultrasonography to rule out DVT, then
antiinflammatory agents and compression stockings 78.7. In a young healthy patient, what percentage of the
cross-sectional area of the pulmonary vascular bed can be
Answer: E. Many patients with superficial thrombophlebitis have
acutely occluded with only minimal symptoms?
a synchronous DVT. Once ruled out, treatment is symptomatic,
A. 10%
with NSAIDs and compression stockings. Ambulation is encour-
B. 20%
aged. Routine anticoagulation is not indicated for superficial
C. 30%
thrombophlebitis.
D. 40%
E. 50%
78.5. A 46-year-old woman presents with pain and swelling of
the right calf. She has a history of tobacco use, Answer: C. Again, this assumes a patient with full cardiopulmo-
emphysema, and hypertension. Medications are albuterol nary reserve and no preexisting disease.
inhaler, lisinopril, 20 mg/day, and oral contraceptives. She
denies pulmonary or cardiac symptoms. Vital signs and 78.8. What percentage of patients with pulmonary embolus
the physical examination are unremarkable except for pain may present with a normal (98%–100%) pulse oximetry
and tenderness to palpation, with minimal swelling of the reading on room air?
right calf. Doppler ultrasonography reveals an isolated calf A. 5%
thrombosis. What is the appropriate management? B. 10%
A. Aspirin therapy with repeat Doppler in 2 to 7 days C. 15%
B. Intravenous fibrinolysis with tenecteplase D. 20%
C. Nonsteroidal anti inflammatory agents and E. 25%
compression stockings
Answer: B. A low oxygen saturation (<95%) increases the prob-
D. Reassurance
ability of pulmonary embolus, but a normal oxygen saturation
E. Systemic anticoagulation
should not dissuade one from the diagnosis.
Answer: E. Approximately 25% of isolated calf DVTs propagate
proximally. Serial Dopplers as surveillance for proximal propaga- 78.9. What percentage of patients diagnosed with pulmonary
tion may be acceptable in healthy ambulatory patients, but full embolus have no apparent clinical risk factor for venous
anticoagulation, as for DVT, would be the safest course of action thromboembolism?
for this patient. A. 10%
B. 20%
78.6. Which of the following statements concerning upper C. 30%
extremity DVTs is true? D. 40%
A. After appropriate treatment, it is rare for symptoms to E. 50%
persist long term after upper extremity DVT.
Answer: E. The point here is that being healthy does not rule out
B. Anticoagulation is not always necessary in upper
the possibility of VTE. Risk factors are best applied to population
extremity DVT.
analysis and are of very limited use when evaluating a single
C. If present, indwelling catheter removal is required for
patient.
successful DVT treatment.
D. Other than catheter-related cases, most occur in
patients who are young and healthy.
E. The rate of pulmonary embolus from axillary vein
DVT is lower than that from the femoral vein.

Descargado para Constanza Vicencio (constanza.vicencio@ua.cl) en Univ Antofagasta de ClinicalKey.es por Elsevier en septiembre 13, 2018.
Para uso personal exclusivamente. No se permiten otros usos sin autorización. Copyright ©2018. Elsevier Inc. Todos los derechos reservados.

Anda mungkin juga menyukai